Constitutional Law MBEs (Practice Exam)

अब Quizwiz के साथ अपने होमवर्क और परीक्षाओं को एस करें!

Assume that technology now exists making it possible for all telecommunications--television, telephone, computers, etc.--to be wireless. However, the crucial question is whether the new wireless utility should be regulated by the federal government or open to marketplace dynamics. The President of the United States engaged in a series of private meetings on this subject. Consumer advocates who are concerned about the price to consumers of such wireless service filed a properly prepared Freedom of Information Act request seeking the notes and minutes of the private meetings the President had on this matter. This Freedom of Information Act request should: (A) Be rejected, because the statements made at these meetings are protected by executive privilege (B) Be rejected, because ordering the President to provide this material could violate separation of powers principles (C) Be granted, because the Freedom of Information Act requires that such material be provided upon the filing of a properly prepared request (D) Be granted, because no person, including the President, is above the law

(A) Be rejected, because the statements made at these meetings are protected by executive privilege Answer (A) is the best answer because it recognizes that the question requires a consideration of executive privilege. While Answer (A) does fail to address the nuances of executive privilege, it still recognizes the central role executive privilege plays in determining whether POTUS must comply with FOIA request. United States v. Nixon, 418 U.S. 683 (1974)

Congress held hearings for several months to investigate reports that agents for the federal Drug Enforcement Agency had planted drug evidence to falsely accuse citizens of engaging in illegal drug activity and had staged armed nighttime raids of the homes of other citizens without probable cause. At the close of the hearings, Congress passed a new federal statute aimed at limiting the investigation powers of the Drug Enforcement Agency. Concerned that the statute might be attacked as making it appear that Congress was "soft on crime", Congress added a non-severable provision to the statute, granting standing to "anyone who wishes to challenge" the statute and providing that the suits would be heard in federal district courts. Wilson, a citizen who supports law enforcement, brought suit in federal district court to challenge the federal statute. The district court should: (A) Dismiss the action, because there is no case or controversy (B) Dismiss the action, because Congress has no constitutional power to confer subject matter jurisdiction in the federal district courts for actions to challenge the constitutionality of a federal statute (C) Hear the action and declare the statute unconstitutional, because it is violative of Article III of the federal Constitution (D) Hear the action and declare the statute constitutional

(A) Dismiss the action, because there is no case or controversy Answer (A) is correct because the doctrine of standing requires that a litigant have a concrete, personal stake in the outcome--the generalized interest of a citizen has never been deemed to be a sufficient basis for conferring standing. Direct, individuated harm--different from citizenry at large is required. The so-called "injury in fact" standard was mandated by the case and controversy requirement of Article III. As a result, it is beyond the power of Congress to confer standing by statute upon a particular class of persons who would not otherwise satisfy the Article III requirement. Congress does not have the authority to expand federal court jurisdiction and grant standing to "anyone who wishes to challenge" the statute. As a private citizen, Wilson would be required to show a direct and personal injury by application of the statute.

In recent years, the city of Santa Ana has been plagued by teenage gang violence. There have been numerous incidents of sporadic shootings and stabbings involving gang members. Unfortunately, even innocent bystanders have fallen victim to this gang warfare. In an effort to alleviate the problem, the city has opened a teenage discotheque where local youths can mingle and dance while under adult supervision. Only teenage boys and girls between the ages of 13 to 19 are permitted to enter the club. When the discotheque first opened, an unexpected problem presented itself. There were a lot more male teenagers frequenting the club than females. This resulted in much harassment and fights which only contributed to the girls staying away. In order to try to correct this situation, the Santa Ana City Council passed an ordinance which provided that any male teenager desiring to enter the discotheque must be accompanied by a female. After this ordinance went into effect, Cody Clever, an 18-year-old youth, was denied admission to the discotheque because he came alone. He then filed suit in federal court challenging the constitutionality of the Santa Ana ordinance. Before his suit came to trial, Cody got married. Also, a Santa Ana city councilwoman has now indicated that she is considering to propose a resolution repealing the ordinance. In light of the foregoing facts, the federal court should: (A) Hear the case (B) Dismiss the case as moot (C) Dismiss the case as unripe (D) Abstain for a short-period of time to see whether the city councilwoman does propose a resolution repealing the ordinance

(A) Hear the case Answer (A) is correct. Under Article III, Section 2 of the Constitution, the "case or controversy" requirement establishes the concept that federal court jurisdiction will not be exercised absent an actual and definite dispute between parties having adverse legal interests. Cody's suit will satisfy this requirement because the unequal treatment by which the ordinance deals with teenage males versus females might adversely affect the legal interests of all teenage males who wish to enter the discotheque, thereby constituting an actual and definite dispute. Answer (B) is incorrect because the case is not moot--a case is moot UNLESS an actual controversy exists at all stages of review. Since a denial of admission could similarly occur to other teenage males trying to enter the discotheque without a female, Cody's lawsuit is not moot.

Congress has recently enacted an act that aims to provide patients with better information about their doctors. The act imposes reporting requirements on all hospitals, both public and private. The reporting obligation is triggered whenever a hospital either (a) receives a complaint against a physician for services provided at the hospital or (b) initiates its own disciplinary proceeding against a physician. Upon either event, the hospital shall, within 60 days of completing internal action regarding the physician, report the following information to the state's physician licensing body: (1) A copy of the complaint filed against the physician or, if none, a written summary of the misconduct leading to the hospital's investigation (2) A written summary of the hospital's actions taken in response to the physician's actual or alleged misconduct (3) A copy of the hospital's written decision, if any, or written summary of the hospital's disposition of the matter. The act directs the United States Department of Health and Human Services to draft regulations prescribing the form for reporting the above information. Failure to make a timely report results in a $1,000 civil fine for the hospital. The act provides that each state's physician licensing authority shall retain all materials gathered from hospitals under the act. Further, on a quarterly basis, each state authority shall forward a copy of the information to the Department of Health and Human Services for inclusion in a national database. Does the act violate the Tenth Amendment? (A) No, because it merely requires the states to comply with a generally applicable law (B) Yes, because it commandeers state hospitals to enforce a federal statute (C) Yes, because it commandeers the state physician licensing authority to enforce a federal statute (D) Yes, because Congress cannot regulate the traditional government function of physician licensing

(A) No, because it merely requires the states to comply with a generally applicable law Answer (A) is correct because a state's own compliance with a generally applicable federal law is NOT commandeering. In Reno v. Condon, 528 U.S. 141 (2000), the Court explained the important distinction between commandeering and taking actions to comply with a generally applicable federal law. Unlike the New York v. United States and Printz cases, the Condon case dealt with a federal law that does not require state legislatures to regulate their citizens and does not require state law enforcement officials to enforce a federal law against their citizens. Since it does neither of these things, it is not commandeering and is instead a generally applicable federal law that the state will need to take certain actions to comply with.

Tish, an avid sports fan, lived in Honolulu. Recently, Select Television, a privately owned and operated company, applied for a permit to provide cable television service to residents of the city. By a vote of 9 to 3, the Honolulu City Council approved Select Television's application and granted a permit for a city franchise. Thereafter, Tish subscribed with Select, which then installed cable service to his home. Tish subscribed with Select because if offered a wide range of sports programs from all across the country. Select installed cable service at Tish's home in February. For the next four months, Tish promptly paid the $25 monthly charge for the cable service. In July, however, Tish was fired from his job as a sushi chef at Benihana Restaurant. Unable to find any work and short of cash, Tish failed to pay Select's bill for the months of July and August. On August 31, Select removed its cable television lines to Tish's home. At no time did Select notify Tish that it would be disconnecting service. However, unknown to Tish, it was Select's policy to terminate any service to any customer who failed to pay his bill for two consecutive months. A short time later, Tish brought suit against Select in federal court seeking an order restoring his cable service. Tish's suit was based solely on the ground that Select's failure to grant Tish a hearing before disconnecting his cable service violated his due process rights under the 14th Amendment. Based on the given facts, the federal court should rule in favor of: (A) Select, because the ownership of a city franchise is not sufficient state action to make the franchisee subject to the 14th Amendment (B) Select, because the ownership of a city franchise allows the franchisee to assert successfully the city's sovereign immunity under the 14th Amendment (C) Tish, because under the 14th Amendment, a person may not be deprived of a valuable property right without notice or an opportunity for a hearing (D) Tish, because a business operating a city franchise is effectively within the public interest, and therefore it is subject to constitutional requirements of procedural due process

(A) Select, because the ownership of a city franchise is not sufficient state action to make the franchisee subject to the 14th Amendment Answer (A) is correct. In Jackson v. Metropolitan Edison Co., 419 U.S. 345 (1974), the Supreme Court held that the actions of a public utility DID NOT involve sufficient state action to subject it to constitutional restraints and that the termination of service by the public utility would not be subject to constitutional review as it did not constitute a state action.

In order to resolve a longstanding territorial dispute with the Poco Indians, Congress enacted a statute conveying 1,000 acres of federal land in southeast Idaho to the Indian tribe. The statute provided that the land in question would be sold to the Poco Tribe for the price of $1.00. After enactment of this statute was made public, many other Indian tribes protested the government's action. In particular, the Alonquonquin Indian tribe, who claimed title to a large tract of federal land in northern Michigan, was angered because no similar land conveyance was made to them. The Aloquonquin Indian tribe brought suit in an appropriate federal district court challenging the constitutionality of this federal statute. Assuming that the Aloquonquin tribe has adequate standing, which of the following statements concerning this suit is most accurate? (A) The court should treat the action of Congress as presumptively valid because the Constitution expressly gives Congress the power to dispose of the territory or other property belonging to the United States (B) The court should treat the action of Congress as presumptively invalid because $1.00 consideration is wholly inadequate to compensate the government for the conveyance of such a large tract of land (C) The court should hold this statute to be constitutional only if substantial evidence demonstrates that the Poco Indian tribe did, in fact, have a valid claim to the land in question (D) The court should hold this statute to be unconstitutional because it denies other Indian tribes the equal protection of the laws

(A) The court should treat the action of Congress as presumptively valid because the Constitution expressly gives Congress the power to dispose of the territory or other property belonging to the United States Answer (A) is correct because Article IV, Section 3 of the Constitution provides that Congress shall have the power to dispose of and make "all needful rules and regulations respecting the territory or other property of the United States". Use of this broad federal property power is subject to congressional discretion NOT subject to state regulation. In light of this federal property power, Congress may validly convey the 1,000 acres of land to the Poco Indians.

A state university adopts a policy prohibiting the admission of any student who is a citizen of a country in which U.S. citizens have been held as hostages for the last five years. Fouad is denied admission to the university because he is a citizen of Iraq, a nation where several Americans are being held hostage. Which of the following is the strongest argument with which to attack the constitutionality of this policy? (A) The state university policy conflicts with the negative implications flowing from the federal government's authority over foreign relations (B) Denial of admission to these foreigners for such past actions of their government is an ex post facto law (C) Admission to a university is so important to a person's economic prospects that it is considered a property interest protected by the 14th Amendment (D) Admission to a state university is a privilege and immunity protected against state infringement by the 14th Amendment

(A) The state university policy conflicts with the negative implications flowing from the federal government's authority over foreign relations Answer (A) is correct. Any state or local law that directly or indirectly conflicts with federal law will be struck down as violative of the Supremacy Clause. The federal government has virtually unlimited power to regulate immigration policy and the conduct of U.S. foreign affairs with other nations. A state government regulation seeking to retaliate for hostage taking of Americans by penalizing the citizens of nations where hostages have been held could seriously undermine the efforts of the federal government to negotiate with these nations. Even when the federal government has not directly prohibited the states from acting in a particular area of regulation, state laws which act as an "obstacle to the accomplishment and execution of the full purposes and objective of Congress have been struck down (Hines v. Davidowitz (1941))

The United States and several European nations have entered a treaty, negotiated by the President and ratified by the Senate, that requires the registration of handguns. The treaty requires each nation to have registration accomplished by specified political subdivisions. In the United States, the treaty requires each state to register these handguns. Several states bring a lawsuit seeking to enjoin the enforcement of this treaty. The states cite the Supreme Court's decision in Printz v. United States, 521 U.S. 898 (1997), which held that the federal government could not force state officials to provide background checks for firearm registration under the Brady Handgun Violence Act. According to the Supreme Court in Printz, the federal government could not force the states to provide their resources to implement a federal program. This commandeering violated the states' sovereignty and was unconstitutional. In their case against the treaty, the states argue the same principles declared in Printz, that this treaty amounts to forcing the states to provide resources to implement a national program and thus violates the states' sovereignty. Which of the following will be the likely outcome of this lawsuit? (A) The states may not prevail, because Missouri v. Holland, 252 U.S. 416 (1920), did not impose a state sovereignty limitation on the treaty power of the federal government (B) The states may not prevail, because the Printz decision involved only background checks rather than the actual act of firearm registration (C) The states may prevail, because Missouri v. Holland has been overruled by the Supreme Court in its recent decisions on state sovereignty, including Printz. (D) The states may prevail, because states can never be forced to comply with treaty provisions unless the states agree to be bound by such treaties

(A) The states may not prevail, because Missouri v. Holland, 252 U.S. 416 (1920), did not impose a state sovereignty limitation on the treaty power of the federal government Answer (A) is correct because in Missouri v. Holland, 252 U.S. 416 (1920), the court held that the protection of migratory birds was a proper subject for a treaty (since birds do not observe national borders) and as an appropriate treaty subject, it falls within Congress' necessary and proper powers. Congress and the federal government may acquire authority under a treaty that they would not otherwise have under Article I. Missouri v. Holland is still good law, for now. It is left to be seen whether the court will protect state sovereignty by saying that the new authority gained under a treaty does not extend to commandeering states in violation of the Tenth Amendment. (B) is incorrect because factual distinction between cases is irrelevant since the question involves commandeering state resources to implement federal programs. (C) is incorrect because Missouri v. Holland has not been overruled. (D) is incorrect because it ignores Supremacy Clause of Article IV. Missouri v. Holland, 252 U.S. 416 (1920)

A state has passed a law providing that all retail sales of automobiles or light trucks to persons located within the state can only be made by licensed retailers with a physical location within the state. Sales in violation of this statute are a crime punishable by jail time and a substantial fine. The law's preamble states: "The distribution and sale of motor vehicles in this state vitally affects the state's economy and the public interest and welfare of its citizens. This Act is an exercise of the state's police power to ensure a sound system of distributing and selling motor vehicles. Licensing only those firms with a physical location within the state serves to prevent fraud and unfair practices. An out-of-state car brokerage firm takes Internet orders for automobiles and light trucks. The firm acts as a broker, taking orders from customers and matching them with car manufacturers or distributors. The manufacturers or distributors then ship the vehicle directly to the purchaser. The firm has no physical retail location in any state and does not sell through any retail outlets. The state's law thus effectively prohibits the firm from selling to consumers in that state, as the firm cannot obtain a license without a physical location within the state. Does the state's law violate the Dormant Commerce Clause? (A) Yes, because the statute discriminates between in-state and out-of-state commerce without adequate reason (B) No, because the state has a valid reason to treat in-state and out-of-state commerce differently (C) Yes, because the statute places an unreasonable burden on interstate commerce (D) No, because the state's interest outweighs the federal interest in interstate commerce

(A) Yes, because the statute discriminates between in-state and out-of-state commerce without adequate reason Answer (A) is correct. The first issue in a Dormant Commerce Clause question is whether the law discriminates between in-state and out-of-state commerce. If the law discriminates, it is per se invalid. These facts are similar to Granholm v. Heald, 544 U.S. 460 (2005) where the court held that because the state could not justify its need to discriminate against out-of-state wine producers, its regulations violated the Dormant Commerce Clause. Answers (C) and (D) are wrong because the statute is discriminatory and the balancing test used for evenhanded laws does not apply.

Congress passes a Water Conservation Act. The act requires all users of water in this country to reduce their consumption by a specified percentage, to be set by a presidential executive order. The act sets forth specific standards the President must use in setting the percentage and detailed procedures to be followed. The provision that allows the President to set the exact percentage is probably: (A) constitutional, because it creates a limited administrative power to implement the statute (B) constitutional, because inherent executive powers permit such action even without statutory authorization (C) unconstitutional, as an undue delegation of legislative power to the executive (D) unconstitutional, because it violates the Due Process clause of the 5th Amendment

(A) constitutional, because it creates a limited administrative power to implement the statute Answer (A) is the best answer. While the President does not have independent power to establish the percentage of water conservation, Congress may delegate its power to do so to the President. This is a legislative action. Congress has the power to regulate water usage under its power to regulate interstate commerce. Second, Congress may delegate certain of its enumerated powers to the President or other administrative agencies. Two requirements for Congress to delegate power (1) the power must be delegable (Congress CANNOT delegate power to declare war, ratify treaties, try cases of impeachment); AND (2) delegation must contain some general guidelines such that a court could determine whether the delegatee had exceeded the authority granted by Congress. Both of these requirements are met under the facts

Congress was concerned about the privacy of Internet communication and banking transactions conducted at automated teller machines (ATMs). In order to increase the protections afforded these electronic transactions and communications, Congress passed the Cyberspace Privacy Act (CPA). Under the CPA, all encryption technology (encryption is the process by which a computer program changes readable text information into an unreadable algorithm or mathematical equation) used to secure Internet and ATM transactions was subject to federal regulation. The CPA strictly regulated the development, use and sale of all encryption software. One provision of the CPA expressly empowered the Office of Defense Trade Controls (ODTC), a federal agency, to regulate the import and export of all encryption technology with foreign countries. Accordingly, the ODTC was given authority to prohibit the export of any encryption system of software if, in the judgment of the ODTC, the export would threaten national security or other foreign policy interests. Professor Gates, a recognized expert on Management Information Systems (MIS), has developed an encryption software program that he was negotiating to sell to a company in France. The ODTC has refused to issue an authorization approving the sale of Professor Gates' encryption software to the French company. Professor Gates filed suit in federal district court challenging the constitutionality of the Cyberspace Privacy Act. The legislation is: (A) constitutional, under the Commerce Clause (B) constitutional, because Congress has the power to regulate for the general welfare (C) unconstitutional, because it deprives Professor Gates of a property right without just compensation (D) unconstitutional, because it usurps the power of the executive branch to conduct foreign affairs

(A) constitutional, under the Commerce Clause Answer (A) is correct. Article I, Section 8 of the U.S. Constitution grants Congress plenary power to regulate interstate commerce and commerce with foreign nations. Since Congress does have the power to regulate commerce with foreign nations, the legislation is constitutional. As such, ODTC would have the authority to regulate the sale of the encryption software program to a foreign country.

The State of Oklahoma has passed a law requiring all commercial restaurants to cook their beef and hamburger meat at a minimum temperature of 165 degrees Fahrenheit. This law, which became known as the "E. Coli Prevention Act" went into effect January 1, 2004. According to an inseverable provision of the law, Oklahoma health inspectors were empowered to levy fines and/or revoke the business permits of any restaurant failing to comply with the health measure. The state legislature enacted this law in order to ensure that beef was properly cooked to kill the E. Coli bacteria. An E. Coli bacteria epidemic has spread across Oklahoma and various sections of the United States, resulting in many deaths to infant children. Health authorities have attributed the E. Coli epidemic to under-cooked beef mostly from fast food restaurant chains. In 2001, Congress had passed a similar bill requiring all commercial restaurants to cook their beef at a minimum temperature of 150 degrees Fahrenheit. After the Oklahoma law went into effect, a state health inspector determined that Burp's Burger King restaurant in Tulsa was cooking its beef at 155 degrees Fahrenheit. As a consequence, the Oklahoma State Health Department moved to revoke Burp's business license as per the provision in the "E. Coli Prevention Act". If Burp's Burder King suit to attack the constitutionality of the Oklahoma "E. Coli Prevention Act", the state law will likely be held: (A) valid, as a reasonable exercise of the state's police power to provide for the health, safety and welfare of its citizenry (B) valid, because health matters are considered a fundamental constitutional right of the citizens of various states (C) invalid, because it conflicts with federal law (D) invalid, because it violates the Due Process Clause of the 14th Amendment

(A) valid, as a reasonable exercise of the state's police power to provide for the health, safety and welfare of its citizenry Answer (A) is the best answer. The Supremacy Clause provides that when Congress tends to occupy a given field (preemption) or where an actual, direct conflict between a federal law and a state law exists, the state law will generally be invalidated. HOWEVER, in cases involving health and safety regulations where the state imposes additional regulations to the existing federal regulations, there is no preemption because regulation of health and safety matters is primarily and historically a matter of local concern (Hillsborough County, Florida v. Automated Medical Laboratories, Inc. 471 U.S. 707 (1985))

The State of North Durango enacted a statute prohibiting any motor vehicle traveling within the state to have window tinting or glass coating of any kind. The bill, known as the "Window Tint" Act, passed the state legislature at the urging of state and local law enforcement agencies who argued that tinted windows prevented them from observing interior car activity. Most citizens supported the "Window Tint" Act, especially after a North Durango state trooper was gunned down and killed by an occupant in a window tinted limousine. The trooper was unable to see that his assailant was armed and dangerous when he approached the vehicle for a speeding infraction. Luxury Limousine Company operates a limo service in the neighboring State of South Durango. Luxury has a fleet of 68 limos, all of which have tinted windows. Each year Luxury makes thousands of trips into North Durango to transport passengers to Durango International Airport (situated 10 miles from the South Durango border) and other destinations. Since all its limos are manufactured with tinted windows, Luxury will incur a great expense to specially order limos without tinting. Luxury brings suit to challenge the constitutionality of the North Durango "Window Tint" Act. Assuming that Luxury has proper standing to assert such an action, which of the following is Luxury's strongest constitutional argument to invalidate the aforesaid statute? (A) Because window tinting is permitted on vehicles in neighboring states, this law denies Luxury the equal protection of laws whenever its limos operate within North Durango (B) Because this law burdens interstate commerce by prohibiting all vehicles with window tinting from entering the state, this law violates the Commerce Clause (C) Because window tinting on vehicles is legal in South Durango, but illegal in North Durango, this law violates the Contract Clause by preventing Luxury from fulfilling its obligation to transport passengers into North Durango (D) Because interstate travel is a fundamental right that may not be burdened by state law, it violates Luxury's substantive due process rights by arbitrarily and unreasonably regulating economic activity

(B) Because this law burdens interstate commerce by prohibiting all vehicles with window tinting from entering the state, this law violates the Commerce Clause Answer (B) is correct. Article I, Section 8 gives Congress a plenary federal commerce power that is held concurrently with the states, which may freely govern matters which to not require uniform national regulation. For a state regulation affecting interstate commerce to pass judicial scrutiny, the statute must be found reasonable and nondiscriminatory upon balancing the benefit to the state against the burden imposed on interstate commerce such that no less restrictive alternative means of regulation is available. Where a state statute may pose a discriminatory or undue burden on out-of-state interests, SCOTUS applies a balancing test (Pike v. Bruce Church). Luxury's strongest argument is that the statute effectively prevents Luxury from doing business in North Durango. Luxury's strongest constitutional argument to invalidate the statute would be the Commerce Clause challenge presented in Answer (B).

Congress enacted a statute providing that persons may challenge a state energy law on the ground that it is in conflict with the federal Constitution in either federal or state court. According to this federal statute, any decision by a lower state court upholding a state energy law against a challenge based on the federal Constitution may be appealed directly to the U.S. Supreme Court. The provisions of this statute that authorize direct U.S. Supreme Court review of specified decisions rendered by lower state courts are: (A) Constitutional, because congressional control over questions of energy usage is plenary (B) Constitutional, because Congress may establish the manner by which the appellate jurisdiction of the U.S. Supreme Court is exercised (C) Unconstitutional, because they infringe the sovereign right of states to have their supreme courts review decisions of their lower state courts (D) Unconstitutional, because Article III does not give the Supreme Court the power to directly hear such appeals

(B) Constitutional, because Congress may establish the manner by which the appellate jurisdiction of the U.S. Supreme Court is exercised Answer (B) is correct because it correctly states the applicable rule of law by which Congress may establish Supreme Court appellate review of specified lower state court decisions. Under Article III, Section 2, Congress has the power to regulate the Supreme Court's appellate jurisdiction. Congress' power to regulate broadly the scope of appellate jurisdiction was upheld in Ex Parte McCardle, 74 US. 506 (1868).

You are still working for our friend, the state legislator. This time she has asked to review proposed legislation establishing a mandatory vaccination program for all state citizens to combat a deadly virus that spreads quickly and has caused epidemics in other countries. An available vaccine can prevent the spread of the disease, and mandatory vaccination programs have been used by other nations in geographical proximity to the pandemic to eradicate the virus in their populace. The legislation thus proposes substantial fines and penalties against those who refuse to be vaccinated, and there are no exceptions to the vaccination requirement. The legislator wants to know if the mandatory vaccination statute would withstand constitutional scrutiny under the United States Constitution. What would you tell her? (A) It is likely that the statute will pass constitutional scrutiny because it does not infringe on a fundamental right or protected liberty interest (B) It is likely that the statute will pass constitutional scrutiny because the liberty interest in refusing medical treatment is outweighed by the compelling governmental interest in eradicating the virus (C) It is unlikely that the statute will pass constitutional scrutiny because it limits the freedoms of individuals to make personal choices without any legitimate justification (D) It is unlikely that the statute will pass constitutional scrutiny because it has no exemptions for religious objections to vaccinations

(B) It is likely that the statute will pass constitutional scrutiny because the liberty interest in refusing medical treatment is outweighed by the compelling governmental interest in eradicating the virus Answer (B) is correct because the court has long recognized that compulsory vaccination programs are constitutional when necessary for public health. Here, based on the experience with the vaccine in other nations, and the threat from the virus that is both deadly and spreads quickly, the state's proposed mandatory vaccination program would not violate the Due Process Clause. (A) is incorrect because refusing medical treatment is a fundamental right or protected liberty that will receive more than rational basis review. (D) is incorrect because religious exemptions are not required for religiously neutral laws of general applicability. Jacobsen v. Massachusetts, 197 U.S. 11 (1905) Cruzan v. Director, Missouri Dept. of Health, 497 U.S. 261, 278 (1990) Washington v. Glucksberg, 521 U.S. 702, 720 (1997) Employment Division v. Smith, 494 U.S. 872

Assume that we are in the future and a former employee of the Department of Defense has sued the sitting President of the United States for money damages. This employee asserts that he was fired in violation of the federal whistleblower statute, claiming that he was dismissed after bringing to the President's attention kickback schemes that fraudulently increased the cost of military equipment. In reviewing a motion to dismiss this lawsuit filed by the President, the lower federal court: (A) Should dismiss the lawsuit against the President because the Secretary of Defense was the proper defendant, not the President (B) Should dismiss the lawsuit because the President has absolute immunity from suit for money damages for conduct while in office (C) Should not dismiss the lawsuit because the President does not have absolute immunity from a suit for money damages for conduct while in office if the suit alleges intentional violation of law (D) Should not dismiss the lawsuit because Congress has authorized the cause of action in passing the whistleblower statute

(B) Should dismiss the lawsuit because the President has absolute immunity from suit for money damages for conduct while in office Answer (B) is correct because it is well established that the President is absolutely immune from a suit for money damages for conduct in office--this applies to sitting and past Presidents. Nixon v. Fitzgerald, 457 U.S. 731 (1982)

In your capacity as a state legislative aide, your legislator asked you to review proposed legislation. This proposed state legislation, a response to the so-called medical malpractice crisis, caps the total amount of damages an injured patient can recover from a physician for medical malpractice (including necessary medical expenses, lost wages, lost earning capacity, pain and suffering) at $1 million, regardless of the severity of the harm to the patient from the malpractice. The accompanying statement to the proposed legislation states that a firm cap on the amount of damages that can be awarded in medical malpractice claims will tend to lower malpractice rates as insurance companies will have an upper limit of liability, which will result in more affordable health care in the state. The legislator has asked you to comment on the constitutionality of this proposed legislation under the United States Constitution. What will you tell her? (A) The Due Process Clause of the Fourteenth Amendment has, since 1937, been interpreted to prohibit only procedural irregularities, but to have no substantive content. Therefore, the proposed legislation would not violate the Due Process Clause of the Fourteenth Amendment. (B) Since 1937 the Supreme Court has predominately used the rationality test in reviewing whether legislation violates the Due Process Clause of the Fourteenth Amendment. Since the state has an interest in ensuring affordable health care by lowering medical malpractice rates, its purpose is legitimate and the method proposed by the legislation--a cap on damages--rationally relates to that purpose. Thus the Supreme Court is likely to uphold the constitutionality of this legislation (C) This legislation would violate the fundamental right of privacy protected by the Due Process Clause because it violates personal autonomy. (D) This legislation violates the fundamental right of access to the court protected by the Due Process Clause by limiting the amount that can be recovered in a medical malpractice suit

(B) Since 1937 the Supreme Court has predominately used the rationality test in reviewing whether legislation violates the Due Process Clause of the Fourteenth Amendment. Since the state has an interest in ensuring affordable health care by lowering medical malpractice rates, its purpose is legitimate and the method proposed by the legislation--a cap on damages--rationally relates to that purpose. Thus the Supreme Court is likely to uphold the constitutionality of the legislation. Answer (B) is correct because the Due Process Clause does have content and is not limited to issues of procedural fairness but the Supreme Court has been reluctant to find new substantive rights contained with the Due Process Clause and since Griswold v. Connecticut, 381 U.S. 479 (1965) has done so in very few cases. Griswold v. Connecticut, 381 U.S. 479 (1965)

During the 1984 Summer Olympic Games in Los Angeles, Algerian terrorists captured members of the U.S. volleyball team and threatened to kill them unless President Regan released three fellow Algerians being held in prison in California. The Algerians who were in prison had been prosecuted and convicted in a California state court for kidnapping. President Reagan realized that if the volleyball team was murdered, it would destroy any chance the United States had of winning a gold medal in that competition. Under intense pressure from the U.S. Olympics Committee, President Reagan decided to make an exception to his inflexible stand on international terrorism. He issued a presidential pardon for the Algerian prisoners and ordered Governor Dukemejian to release them. However, the governor refused to honor the Presidential order. Which of the following is the strongest constitutional ground for the governor's refusal to release prisoners? (A) The President's power as Commander-in-Chief does not extend to persons held in state custody (B) The President has the constitutional power to grant pardons or reprieves only for offenses against the United States (C) Under intergovernmental immunity, the governor acting pursuant to state laws may disregard inconsistent orders by federal officials (D) The President's duty to take care that the laws be faithfully executed required that he honor all terms of a state court conviction

(B) The President has the constitutional power to grant pardons or reprieves only for offenses against the United States Answer (B) is correct. The President can only grant pardons or reprieves for federal offenses, not state offenses. The governor has pardon power over persons convicted of state crimes and the Algerian prisoners were subject to a California state criminal prosecution.

In the wake of a string of terrorist attacks throughout Europe, Congress recently issued the following declaration: "Whereas, terrorism remains a method of attack by which enemies of the United States seek to destroy the American way of life by undermining citizens' confidence in their country's ability to protect them. Whereas, terrorism is not perpetrated by any single, nation-state, but rather through groups associated with and supported by sympathetic nation-states. The United States declares war against all persons who perpetrate terrorist acts against the United States and its citizens, wherever they may be found, as well as all person and nations who give aid and comfort to such terrorists." Five members of the United States Army have filed suit challenging the constitutionality of Congress' declaration. These servicepersons allege that they have received deployment orders to Iran that are specifically pursuant to the above declaration. Their suit argues that Congress' power to declare war requires the national legislature to identify a specific nation against which the declaration applies. What is the strongest argument against a federal court hearing this lawsuit? (A) The servicepersons lack standing (B) The lawsuit poses a political question (C) The lawsuit is not ripe (D) The lawsuit is barred by the Eleventh Amendment

(B) The lawsuit poses a political question Answer (B) is correct because political question is the strongest grounds for opposing the plaintiff's lawsuit. Baker v. Carr, 369 U.S. 186 (1962) established that an issue is a political question if any one of six factors is "inextricable from the case at bar". This is likely so for several of the factors. Congress textually commits the power to declare war--Congress has the sole power to declare, the POTUS has the power to wage war and Congress can raise and maintain the military. The Constitution gives no judicially discoverable or manageable standards for judging the sufficiency of a congressional declaration of war.

In September, Hurricane Helga struck the Florida coast with devastating force. Helga was a grade 5 hurricane and caused over $100,000,000 in damage. Florida farmers were especially hit hard by the hurricane, which destroyed over 30% of the state's citrus crops. As a consequence, Congress passed an emergency farm aid bill providing $50,000,000 in subsidies to Florida orange growers to compensate them for economic losses. Clearly, Florida's economy is very much dependent upon the orange industry, which is its largest staple crop. The federal farm aid bill, however, did not provide any compensation to Florida farmers growing crops other than oranges. Pappas is a Florida papaya farmer who suffered substantial economic loss when his entire crop was destroyed by Hurricane Helga. Pappas challenges the constitutionality of the aforementioned law. Is this act of Congress constitutional? (A) Yes, because Congress has exclusive jurisdiction as to the propriety of dispersing federal funds which is not subject to judicial review (B) Yes, because Congress has the power to appropriate funds for the general welfare even though disparate treatment is given to different groups (C) No, because the law violates the equal protection rights of farmers of products other than oranges who are not entitled to governmental subsidies (D) No, because the law denies farmers of products other than oranges their fundamental rights without due process.

(B) Yes, because Congress has the power to appropriate funds for the general welfare even though disparate treatment is given to different groups Answer (B) is correct. Under Article I, Section 8, Congress has plenary power to tax and spend for the general welfare and the Supreme Court has held that Congress is NOT limited to spending only to achieve the specific powers granted in Article I of the Constitution--Congress may spend in any way it believes would serve the general welfare, do long as it does not violate another constitutional provision. (United States v. Butler, 297 U.S. 1 (1936)) Answer (A) is incorrect because Congress' spending power IS subject to judicial review

The President was concerned about a possible trade embargo imposed on U.S. exports to the country of Blekarne because U.S. manufacturers operating in Blekarne were paying wages to Blekarne workers at a rate substantially lower than those paid to U.S. workers performing comparable work. In order to avoid this problem, the President entered into an executive agreement with the leader of Blekarne requiring that any U.S. based company employing workers in Blekarne must pay those workers no less than 80% as much as any U.S. worker employed by the company in the United States performing a similar function. Several U.S. manufacturers complained that this would place them at a gross disadvantage compared to non-U.S.-based companies who did not have to increase their wages for Blekarne workers. The U.S. manufacturers complained to Congress, but Congress was unable to produce legislation satisfactory to a majority of the members. If the executive agreement is challenged in federal court, the court will: (A) refuse to hear the case because matters of foreign policy are nonjusticiable political questions (B) rule that the executive agreement is constitutional as within the powers of the executive branch but is subject to judicial review (C) rule that the executive agreement is unconstitutional since the subject of the agreement requires that a treaty be entered into with the full advice and consent of the Senate (D) rule that the executive agreement is unconstitutional since it violates the Contract Clause rights of U.S. manufacturers

(B) rule that the executive agreement is constitutional as within the powers of the executive branch but is subject to judicial review Answer (B) is correct. As a general rule, matters involving foreign affairs are nonjusticiable and immune from judicial review. However, executive agreements relating to economic matters are generally subject to judicial review. Answer (D) is incorrect because the Federal government is not subject to the Contract Clause

The U.S. Department of the Interior granted Reeves a food and drink concession in a federal park located in the State of Cumberland. Reeves operated this concession for his own profit. The concession was operated in federally-owned facilities, for which Reeves paid rent, and served visiting tourists from all over the county. The legislature of the State of Cumberland, in an effort to equalize state tax burdens, passed a law imposing a rental use tax on all persons occupying real property in Cumberland for which no state real property taxes are paid. Pursuant to the new law, the Cumberland Department of Revenue attempted to collect the state rental tax use from Reeves, based on the estimated fair market rental value of the federal facilities housing his operation. Reeves sued in federal district court to invalidate the state rental use tax as applied to him. The District Court should hold that the state rental use tax in this instance is: (A) valid, because federal property used for propriety purposes is not immune from nondiscriminatory taxation (B) valid, provided the tax does not discriminate against interstate commerce (C) invalid, because this tax interferes with Reeve's right to serve interstate tourists on federal property (D) invalid, because the Supremacy Clause does not permit taxation of any federally owned property

(B) valid, provided the tax does not discriminate against interstate commerce Answer (B) is correct. While the Supremacy Clause prohibits state or local governments from directly regulating or taxing the federal government without its consent, the tax here is not upon the federal government but rather upon a private concessionaire using federal property. Such "property interest taxes" tax the user's right to the use and enjoyment of federal property, although the property itself is exempt from state taxation. So long as the property interest tax is nondiscriminatory, it is not constitutionally prohibited.

The State of Rutherford is known for its coal production. The neighboring State of Chippewa also contains large coal deposits. Chippewa's main coal mining company, however, is in poor financial shape and there are rumors in the industry that it may cease doing business and sell a large quantity of coal at bargain prices. Fearful of an impending glut on the market and the consequent effect on the local coal industry, the Rutherford state legislature enacts a law which prohibits sales of coal within the Rutherford state boundaries at less than $25/ton, which is the current market price. This law is most likely to be held constitutional as applied to which of the following coal vendors? (A) A Chippewa vendor offering coal for sale to the general public in the State of Rutherford at $20/ton (B) A Rutherford vendor offering coal for sale to the U.S. Veteran's Hospital in Rutherford at $20/ton (C) A Rutherford vendor offering coal for sale to a manufacturing plant in Rutherford at $20/ton (D) A vendor located outside of the United States offering coal for sale to the general public in the State of Rutherford at $20/ton

(C) A Rutherford vendor offering coal for sale to a manufacturing plant in Rutherford at $20/ton Answer (C) is correct. States have the power (concurrent with the federal government) to regulate commerce within their own borders. Such regulation will generally be upheld if it is purely local (intrastate) and neither produces a lack of uniformity which is of federal concern nor is already preempted by federal legislation in the area. Since Answer (C) presents a case of purely intrastate commerce, the regulation will likely be upheld.

The House of Representatives recently impeached the Vice President of the United States for allegedly providing false and misleading information to members of Congress during periodic intelligence briefings. Immediately after the House vote on impeachment, the Vice President files suit in the United States District Court for the District of Columbia to enjoin the upcoming trial in the Senate. The Vice President argues that the House impeachment is void because the House procedures violate the Constitution. Specifically, the Vice President argues that impeachment is equivalent of an indictment by a federal grand jury and so the House impeachment procedures must mirror those followed by federal grand juries. Because the House followed evidentiary and other rules that differ substantially from those used by federal grand juries, the impeachment allegedly violated Article I, Section 2, Clause 5. Counsel for the House files a motion to dismiss the Vice President's lawsuit because it poses a non-justiciable political question. The Vice President's lawsuit likely poses a political question because: (A) The issue of the proper procedures for impeachment is textually committed to the House of Representatives (B) Judicial resolution of the issue would interfere with the substantial need for finality in the matter (C) A and B (D) Neither A nor B

(C) A and B Answer (C) is correct because in Nixon v. United States, 506 U.S. 224 (1993), the Supreme Court dismissed the case as raising a political question because (1) the Constitution's text committed the decision to another branch by granting the Senate the "sole power to try all impeachments" which entails the power to determine what procedures to use at trial and (2) the impeachment decision demanded finality--if litigation over the impeachment procedures could create a long period of uncertainty, the lawsuit should be dismissed as a political question Nixon v. United States, 506 U.S. 224 (1993)

The Commonwealth of Newcastle has a vast coastal shoreline. Each year, thousands of boats are manufactured and sold there. In light of its expanded boat industry, Newcastle passed a law which established a Department of Boating to oversee and regulate boat sales within the state. In an inseverable provision, the Newcastle Boating Act also prohibited any person from selling at retail any new or used boat that has not been inspected and approved for safety by a licensed boating inspection station in Newcastle prior to the time the vendor delivers possession to the purchaser. Despite protests from boat manufacturers and dealers, the Boating Act was passed by Newcastle state legislature and signed into law by the governor. Which of the following persons would most likely be able to obtain in an appropriate federal court an adjudication on the merits of the constitutionality of the Newcastle Boating Act? (A) A resident of Newcastle who is considering the possible purchase of an uninspected boat in that state (B) A resident of Newcastle who intends to sell in Newcastle uninspected boats to other residents of Newcastle (C) A boat manufacturer who sells to retailers in Newcastle under ambiguous contracts that might require the manufacturer to reimburse retailers for the cost of such inspections (D) A taxpayer of the state of Newcastle who believes the cost of enforcing this statute is an utter waste of the state funds

(C) A boat manufacturer who sells to retailers in Newcastle under ambiguous contracts that might require the manufacturer to reimburse retailers for the cost of such inspections Answer (C) is correct because a boat manufacturer currently selling to Newcastle retailers most likely has standing to challenge the Newcastle Boating Act. Since this manufacturer is presently selling to Newcastle retailers, the likelihood of his incurring reimbursement costs for the boat inspections is more distinct and actual than that of any plaintiff in the other given answer choices. Furthermore, an adjudication in the favor of the boat manufacturer will redress the claimed injury, i.e., establishing a causal connection.

A state recently passed a law that reduced the speed limit on its state highways from 70 to 60 miles per hour. Each of the bordering states has a speed limit of 70 miles per hour on their state highways. A national trucking company filed a federal lawsuit claiming that the new state speed limit violates the Dormant Commerce Clause because it forces interstate traffic to slow down when it enters this state. This state speed limit is most likely: (A) Unconstitutional because the state has discriminated against out-of-state traffic for the benefit of in-state traffic (B) Unconstitutional because the state's legitimate interest in traffic safety does not clearly exceed the speed limit's burden on interstate commerce (C) Constitutional because the speed limit's slight burden on interstate commerce does not clearly exceed the state's legitimate interest in traffic safety (D) Constitutional because the safety of state highways is a purely local matter that may be regulated by the states as they please

(C) Constitutional because the speed limit's slight burden on interstate commerce does not clearly exceed the state's legitimate interest in traffic safety Answer (C) is correct because it notes that the state law strikes a permissible balance between slight burden on interstate commerce and significant benefit to in-state traffic safety. The facts parallel the Kassel v. Consolidated Freightways Corp., 450 U.S. 662 (1981) case but these facts are distinguished in that the present facts do not demonstrate a great economic burden, there is an overwhelming evidence of the safety benefits and the law is non-discriminatory in that it applies evenly to all and does not confer a benefit to in-state residents. Kassel v. Consolidated Freightways Corp., 450 U.S. 662 (1981)

Booker is an American citizen who, as part of his business, imports books and other reading materials from other countries. This work entails periodic trips abroad to attend book fairs and other trade shows. Since 2000, Booker has attended a book fair in a foreign country. To travel to this foreign country, he must obtain a license from the United States Department of the Treasury; the Department has issued him a license each year since 2000. In December 2015, Booker applied for licenses to attend the book fair for each of the next five years. The next fair is to be held on February 12-15, 2016. On January 2, 2016, Booker received a letter from the Department requesting further information to process his applications. Booker sent this information on January 6. On January 31, the Department had not acted on his applications, and inquiries with the Department only revealed that his applications were "open and pending". On February 1, 2016, Booker filed a lawsuit in federal district court seeking a writ of mandamus ordering the Department to act on his license application for the 2016 book fair. The Department moved to dismiss his lawsuit. On March 1, while its motion was pending, the Department granted him a license to travel to the 2017 book fair. What is the best ground for dismissing the claim regarding the Department's inaction on his license application for the 2016 book fair? (A) He lacks standing to challenge the Department's inaction on his 2016 license request (B) HIs challenge to the Department's inaction on his 2016 license request is not ripe (C) His challenge to the Department's inaction on his 2016 license request is moot (D) Eleventh Amendment immunity bars his federal court lawsuit against the Department

(C) His challenge to the Department's inaction on his 2016 license request is moot Answer (C) is correct because the passage of time has made the 2016 license application a moot issue. Booker has requested only a writ of mandamus, which is an order that the Department issue the license. The 2016 license covers travel dates that have already passed. Thus a writ of mandamus ordering the issuance of the 2016 travel license would have no effect and the claim is moot.

Congress recently enacted an act regarding providing legal services to the poor. The act establishes a poverty law office in every state that is responsible for representing indigent clients in family law matters, such as divorce, child custody and child support matters. The act funds these new offices by imposing an occupation tax on every attorney licensed to practice in a state, territory, or possession of the United States. To facilitate collection of the attorney occupation tax, the act requires every firm or other organization, public or private, that employs lawyers to register with the Department of Labor, and to report quarterly the names and wages of each lawyer then employed. The act places a similar reporting requirement on solo practitioners. A national association of state attorney generals recently spoke out against the law, arguing that the act's registration requirement and occupation tax violate the residue of state sovereignty protected by the Constitution and Bill of Rights when applied to attorneys employed by the state. Does the act's registration requirement violate the Tenth Amendment? (A) Yes, because it commandeers the state legislature into making laws that bring the state into compliance with the registration requirement (B) Yes, because it commandeers the state agencies that employ lawyers into complying with the registration requirement (C) No, because the act merely requires states to comply with a generally-applicable registration requirement (D) No, because the practice of law is an economic activity with a substantial effect on interstate commerce

(C) No, because the act merely requires states to comply with a generally-applicable registration requirement Answer (C) is correct because commandeering only occurs when Congress directs the manner in which States regulate private parties, NOT when Congress applies a general regulation to both states and private parties alike. The act is a generally applicable law that merely asks the states, as one of many organizations that employs lawyers, to register their own lawyers. Answer (D) is incorrect because it is an incomplete statement of the law--even if Congress may reach the regulated activity under the Commerce Clause, Congress may not do so by commandeering states to regulate. South Carolina v. Baker, 485 U.S. 505 (1988) New York v. United States, 505 U.S. 144 (1992)

A county ordinance establishes several mandatory provisions that must appear in any contract between the county and general contractor working on a county project. One provision states that the general contractor will receive a $1,000 bonus for each county resident it employs to work on a county-funded project. The county enacted this ordinance to address the runaway unemployment in its region of the state, which has caused loss of business for local merchants, reduced sales tax revenue, and prompted middle class families to move to other parts of the state. Does the ordinance's resident hiring bonus violate the Dormant Commerce Clause? (A) No, because the Dormant Commerce Clause does not apply to municipal ordinances (B) Yes, because the ordinance discriminates between in-state and out-of-state commerce without justification (C) No, because the county is acting as a market participant (D) Yes, because the county's contract with the general contractor imposes a discriminatory restraint on downstream conduct.

(C) No, because the county is acting as a market participant Answer (C) is the best answer because the ordinance fits within the market participant exception. Under the market participant exception, a state may contract on any terms it wishes when participating within a market--even terms that discriminate against out-of-staters without violating the Dormant Commerce Clause (Reeves, Inc. v. Stake, 447 U.S. 429 (1980)). However, the court did recognize that the market participant exception applies ONLY TO THE MARKET IN WHICH THE STATE ACTUALLY PARTICIPATES (South-Central Timber Development, Inc. v. Wunnicke, 467 U.S. 82 (1984)). In this present case, the county should be treated as a market participant in the market for hiring the contractors employees and its preference is not a downstream restraint (Massachusetts Council of Construction Employers, Inc. 460 U.S. 204 (1983)).

Congress was very concerned about the rise in gun-related deaths and injuries that had been occurring in schools. To combat the problem, Congress passed a statute which required the states to enact legislation that made it a crime to sell, possess, or transport any firearm within 100 yards of any school, if that firearm had been purchased or sold through or using any instrumentality of interstate commerce. Is the statute constitutional? (A) Yes, because the statute will permit a case-by-case review of the adequacy of an interstate commerce connection between the permissible objectives of the federal statute and any individual criminal prosecution (B) Yes, because the statute is a proper attempt to regulate interstate commerce in its inseverable aggregates (C) No, under the separation of powers doctrine, Congress cannot commandeer the states to enact and enforce legislation (D) No, the connection between interstate commerce and the purpose of the regulation is too remote to justify the exercise of the Commerce Clause power

(C) No, under the separation of powers doctrine, Congress cannot commandeer the states to enact and enforce legislation Answer (C) is correct. Congress CANNOT "commandeer the legislative process of the states by directly compelling them to enact and enforce a federal regulatory program". The Court has held that there are limits to Congress' right to interfere with the states' lawmaking process and Congress violates the 10th Amendment if and when it exceeds these limits. (New York v. United States, 505 U.S. 144 (1992)

Referring to the facts in the preceding Question 11, assume that allegations have been made that members of the telecommunications industry have paid money to members of the President's administration in order to receive favorable treatment. These allegations are under grand jury investigation by an independent prosecutor. The minutes of the President's private meetings discussed in the above Question 11 are subpoenaed by the grand jury. The President has sought to quash this grand jury subpoena on the grounds of executive privilege, stating that the minutes contain national security secrets. In ruling on this motion to quash the subpoena, a court: (A) Should quash the subpoena on separation of powers principles (B) Should quash the subpoena because the information requested contains national security secrets (C) Should quash the subpoena unless the prosecutor can show that the material subpoenaed is essential to do justice in the case being investigated (D) Should quash the subpoena because the President has an absolute executive privilege

(C) Should quash the subpoena unless the prosecutor can show that the material subpoenaed is essential to do justice in the case being investigated Answer (C) is correct because it clearly follows the Court's holding in United States v. Nixon, 418 U.S. 683 (1974). After balancing the competing interests, the Court found the best method to protect President's need for confidentiality and teh public's need for fair administration of criminal justice was to grant the President a presumptive privilege and then require the prosecutor to establish that the materials were "essential to the justice of the [pending criminal] case". United States v. Nixon, 418 U.S. 683 (1974)

Assume the President has, over a period of years, negotiated with many other countries on various aspects of global environment protection. These aspects include regulation of ozone emissions, discharge of sewage into the ocean, etc. The President fears that if these agreements were submitted to the Senate for approval, it would not approve these measures. The President, therefore, signs an executive agreement between the United States and these other countries, committing the United States to take certain actions regarding aspects of global environment protection, including regulating ozone emissions. Several years before the executive agreement on regulation of ozone emissions was signed, a federal statute went into effect that permitted much greater ozone emissions than does the executive agreement. (A) The executive agreement on ozone emission takes control over the federal statute on ozone emissions because it was signed after the federal law went into effect (B) The executive agreement should be held constitutional because the President has authority to commit the United States in its foreign affairs through executive agreements (C) The executive agreement should be declared unconstitutional under separation of powers principles because the President does not have the authority to unilaterally override domestic federal law (D) The executive agreement should be declared unconstitutional under separation of powers principles because there is no indication that the President has to take this action in order to avoid a crisis

(C) The executive agreement should be declared unconstitutional under separation of powers principles because the President does not have the authority to unilaterally override domestic federal law Answer (C) is correct because in Medellin v. Texas, 552 U.S. 491 (2008), the Supreme Court held that the President could not unilaterally create domestic law even in pursuit of important foreign policy goals. Medellin v. Texas, 552 U.S. 491 (2008)

The State of Midwest recently enacted legislation authorizing the state's tax collection agency to issue refunds of state tax payments in the form of "Midwest Bucks". Midwest Bucks are vouchers good for the purchase of various consumer goods at special state retail outlets. In practice, one Midwest Buck is slightly greater in buying power than one dollar, since the special state retail stores are exempted from paying state retail sales tax. A significant commerce in Midwest Bucks has sprung up independent of the state retail stores; many people are willing to accept Midwest Bucks in exchange for services or property, knowing that they can use them to purchase goods at the state stores. The U.S. government has instituted an action in federal court to declare the Midwest Bucks legislation as unconstitutional. Which of the following is the strongest argument supporting a judicial declaration that the challenged statute is unconstitutional? (A) The statute unduly burdens commerce (B) The statute violates the Privileges and Immunities Clause of Article IV (C) The statute is inconsistent with the express power of the federal government to coin money and regulate its value (D) The authority to tax, and the concomitant authority to issue tax refunds in any legitimate manner, in inherent in the powers reserved to the states by the 10th Amendment.

(C) The statute is inconsistent with the express power of the federal government to coin money and regulate its value Answer (C) is correct because under Article I, Section 8 of the Constitution, Congress is granted the exclusive power to coin money. Any attempt by state or local governments to create a substitute or competitive currency will be struck down as violative of the exclusive federal power.

Sadly the United States suffers another attack by terrorists who seized a nuclear power plant and almost caused a deadly release of radiation. They were stopped and captured before achieving success. One of the terrorists is a United States citizen, and the others are foreign nationals. The President designates all as enemy combatants and orders that they be held at a United States military facility somewhere in the United States. The order denies them a hearing before a tribunal that would determine specific facts in support of continued detention. Each detainee files for a writ of habeas corpus in federal district court seeking release unless the government can show some basis for their continued detention. The President's order denying a hearing to all captured individuals is: (A) Constitutional because the President has inherent power to protect and defend the United States, which power includes this order. (B) Constitutional because the need to stop terrorist activity justifies taking action that is extraordinary under the Constitution (C) Unconstitutional because separation of powers principles require that a citizen have access to the courts to challenge the detention (D) Unconstitutional because the President's power under Article II of the Constitution is limited by the text of the Constitution

(C) Unconstitutional because separation of powers principles require that a citizen have access to the courts to challenge the detention Answer (C) is correct because it best takes account of the Court's decisions in Hamdi v. Rumsfeld, 542 U.S. 507 (2004) and Boumediene v. Bush, 553 U.S. 723 (2008). In Hamdi, the Supreme Court held that even if a detention involves military and national security matters, separation of powers principles require some judicial role when individual liberties are at stake. In Boumediene, the Supreme Court held that aliens detained in Guantanamo Bay had the right to seek habeas corpus even though Congress had enacted legislation purporting to strip the federal courts of habeas jurisdiction in cases involving designated enemy combatants. The President CANNOT deny detainees a hearing on the validity of their detention Hamdi v. Rumsfeld, 542 U.S. 507 (2004) Boumediene v. Bush, 553 U.S. 723 (2008)

Congress has passed the following statute: "A person who knowingly discharges a firearm at a person from a motor vehicle while traveling along a public roadway shall be punished by imprisonment for not more than 25 years, and if death results, shall be punished by death or imprisonment for life" Does Congress have power under the Commerce Clause to enact this statute? (A) No, because the regulated activity is neither economic nor commercial (B) No, because Congress may not aggregate the economic effects of violent crime (C) Yes, because Congress is regulating a channel of interstate commerce (D) Yes, because drive-by shootings substantially affect interstate commerce by discouraging interstate travel

(C) Yes, because Congress is regulating a channel of interstate commerce Answer (C) is correct because Congress should have an interest in keeping roadways free from violent crimes as channels of interstate commerce. While arguable, Answer (D) is weaker than Answer (C) because it conflicts with the court's reasoning in United States v. Morrison--the court will refuse to aggregate an activity's impact on interstate travel if the activity is not commercial or economic (gender-motivated violence in Morrison and drive-by shootings in this question) United States v. Lopez, 514 U.S. 549 (1995) United States v. Darby, 312 U.S. 100 (1941) Heart of Atlanta Motel, Inc. v. United States, 379 U.S. 214 (1964) United States v. Morrison, 529 U.S. 598 (2000)

A state has the following tolling provision for its statutes of limitations: "The applicable statute of limitations shall be suspended for a period that a person is outside this state" The provision was enacted in 1880 in response to the Supreme Court's decision in Pennoyer v. Neff, 95 U.S. 714 (1878). Pennoyer held that a state could only assert personal jurisdiction over civil defendants served with process while physically present in the state. The state legislature feared that a potential civil defendant might flee the state to avoid service or process, returning only after the statute of limitations had run. In doing so, the prospective defendant could rob injured state residents of their rightful, legal compensation. To prevent this abuse, the state passed the above tolling provision. Does the state's tolling provision violate the Dormant Commerce Clause? (A) Yes, because the statute discriminates between in-state and out-of-state commerce without jurisdiction (B) No, because the statute treats in-state and out-of-state commerce evenhandedly (C) Yes, because the statute substantially burdens interstate commerce without advancing a legitimate state interest (D) No, because the state interest outweighs any burden on interstate commerce

(C) Yes, because the statute substantially burdens interstate commerce without advancing a legitimate state interest Answer (C) is the best answer.

Congress was concerned about the growth of commerce taking place over the Internet. In Congressional hearings, business interests convinced Congress that the growth of Internet commerce would be seriously restricted unless limits were placed on the ability of plaintiffs to file suit in a distant federal court against out-of-state website operators who lack the financial resources to answer suit in every state where an Internet user might be located. After the hearings had ended, the members of Congress agreed that the growth of the Internet and availability of state courts for redress of Internet grievances had rendered federal court suits based on diversity of citizenship an unnecessary and dangerous impediment to the growth of Internet commerce. Thus, the Congress, over the veto of the President, enacted legislation which completely barred all suits in federal court based on diversity of citizenship jurisdiction. This legislation is: (A) unconstitutional because it violates the doctrine of separation of powers by usurping the jurisdiction of the federal judiciary mandated by Article III of the Constitution (B) unconstitutional because it violates 5th Amendment Due Process (C) constitutional because it is within Congress' power to provide the jurisdiction of inferior federal courts consistent with Article III (D) constitutional as a necessary and proper means of protecting the general welfare

(C) constitutional because it is within Congress' power to provide the jurisdiction of inferior federal courts consistent with Article III Answer (C) is correct. Under Article III, Section I, Congress has discretion to create lower (inferior) federal courts. Therefore, Congress also has the power to determine their jurisdiction. (Sheldon v. Sill, 49 U.S. 441 (1850))

The Florida legislature has passed a law banning the sale of phen fen, a popular weight loss pill, within the state due to widespread reports that the drug caused users to suffer cardiac arrest. The Florida statute prohibited the sale and distribution of phen fen either by prescription or by over-the-counter sales. Violation of the statute was proscribed to be a misdemeanor punishable by a $1,000 fine. After the Florida law was enacted, the Food and Drug Administration (FDA), a federal agency, issued an administrative order allowing the sale of phen fen by prescription only. Eli Einhorn is a registered pharmacist who owns and operates a pharmacy in Miami Beach. Sophie Sadler, a Miami Beach resident, was given a prescription for phen fen by her physician. She presented the prescription to Eli who dispensed phen fen to her. Thereafter, Eli was charged with violating the Florida statute prohibiting the sale of phen fen. As Eli awaited trial, his attorney filed suit challenging the constitutionality of the Florida statute. The court will likely hold that the statute is: (A) valid, because the state law was enacted prior to the administrative order (B) valid, because a state law takes precedence over a conflicting administrative order (C) invalid, because an administrative order of a federal agency supersedes a conflicting state law (D) invalid, because the FDA has plenary power to regulate the sale and distribution of drugs within the United States

(C) invalid, because an administrative order of a federal agency supersedes a conflicting state law Answer (C) is the best answer. As a general rule, the actions of federal agencies may preempt state law. According to Nowak, "when Congress gives an administrative agency discretion to regulate a field of commercial activity, the agency's decision to preempt state regulations should be upheld"

Paul recently left his job in Wildwood, a rural community in the State of Calzona, and moved his family to Industria in search of new career opportunities there. Industria is located in an urbanized metropolitan area of Calzona, which has been experiencing tremendous employment and residential growth as of late. When Paul informed his insurance carrier, Good Neighbor Insurance Company, of the change of residence, he was notified that his auto and home insurance rates would increase by 300% due to the large number of traffic accidents and theft claims occurring in Industria. The insurance industry is highly regulated by Calzona law, including the licensing and solvency of all insurance companies operating within the state. There are no state laws regulating rates for insurance practices of the companies. If Paul brings suit in Calzona state court claiming the rate increase is discriminatory in violation of the Equal Protection Clause and the court rules against him, the most likely reason will be because: (A) states have plenary power to regulate the insurance industry within their own state boundaries (B) there is a valid reason to discriminate (C) the Equal Protection Clause is not applicable (D) the Equal Protection Clause is satisfied

(C) the Equal Protection Clause is not applicable Answer (C) is correct. An Equal Protection Clause claim requires state action. The rate setting policies of private insurance companies do not involve state action. The Calzona laws regulate only the licensing and solvency of insurance companies, not the premiums charged. State licensing of a private entity is not sufficient to render the discriminatory practices of the private entity "state action" within the meaning of the 14th Amendment (Moose Lodge v. Irvis (1972))

President Clinton was a longtime admirer of Crispus Attucks, one of the five men killed in the Boston Massacre. As his last official act as President, Clinton issued an executive order making January 2nd, Crispus Attucks' birthday, a paid national holiday for federal executive employees. Congressional legislation currently in effect designates 10 paid national holidays for federal employees. The applicable federal law neither authorizes nor prohibits the President's action. Clinton's executive order extends only to members of the executive branch of the federal government. Faith Hall is a recent Harvard Law School graduate who has been hired by Supreme Court Justice Scalia as a law clerk. Faith is upset because she is required to work on January 2nd and President Clinton's order does not extend to federal judicial employees. Faith brings suit in federal court challenging the validity of the executive order. The executive order is: (A) constitutional, because it is not expressly prohibited by any prior act of Congress (B) constitutional, because the President as chief executive has plenary power over employment matters of executive employees (C) unconstitutional, because it is an executive act that usurps congressional authority (D) unconstitutional, because it discriminates against federal employees outside the executive branch who are not given a paid holiday

(C) unconstitutional, because it is an executive act that usurps congressional authority Answer (C) is correct. A treaty or act of Congress will supersede and take precedence over an inconsistent executive agreement or order. Executive agreements and orders are effective and binding as long as they do not supersede inconsistent provisions of earlier acts of Congress. The President's action is clearly in contravention with an earlier act of Congress.

In which of the following instances would the 11th Amendment most likely bar the court action? (A) Citizen of New York files action against New York Attorney General in U.S. District Court seeking an injunction against enforcement of a New York policy that alleges violations of federal law (B) Citizen of California files action against Governor of Nevada in a Nevada state court seeking an injunction against enforcement of Nevada statute that alleges violation of federal law (C) Citizen of Illinois sues City of Chicago in U.S. District Court for alleged violations of federal law (D) Citizen of Florida sues the Florida Board of Regents in U.S. District Court for damages and injunction for alleged violations of federal law

(D) Citizen of Florida sues the Florida Board of Regents in U.S. District Court for damages and injunction for alleged violations of federal law Answer (D) is correct. 11th Amendment bars suits against states and arms of the state (including state agencies) from being filed in federal court. Answer (A) is incorrect because lawsuits filed in federal court against state officials are not barred by 11th Amendment, as long as plaintiff is not seeking damages to be paid by the state itself (Ex Parte Young, Edleman v. Jordan) Answer (B) is incorrect because the complaint is filed in a state court. Answer (C) is incorrect because suits filed in federal court against the political subdivisions (e.g. cities, counties, etc.) of a state are not barred by 11th Amendment.

A U.S. Air Force military plane mistakenly shot down a commercial airplane operated by the country of Ubiskagaski. The U.S. Air Force officials incorrectly believed that the jetliner was an enemy plane that was about to attack a U.S. military installation. There were no survivors of the crash. Families of the deceased passengers have hired lawyers in the United States to sue the U.S. government in federal district court for wrongful death. The federal court should: (A) Dismiss the action, because it is nonjusticiable since it deals within the area of foreign affairs (B) Dismiss the action, because it is nonjusticiable since only U.S. citizens and resident aliens may bring suit in federal court (C) Dismiss the action, because the plaintiffs lack standing under Article III of the Constitution (D) Hear the case and decide its merits, if there is statutory authority to allow foreigners to sue the U.S. government

(D) Hear the case and decide its merits, if there is statutory authority to allow foreigners to sue the U.S. government Answer (D) is correct. The United States, as a sovereign, is immune from being sued unless it has waived its sovereign immunity by statute. (United States v. Thompson, 98 U.S. 456, 489 (1878) Consent to sue the federal government must be clearly given, expressly and implicitly. Answer (A) is incorrect because sovereign immunity is not a nonjusticiable issue and is a proper subject for judicial review. Answer (B) is incorrect because Congress, by statute, can authorize nonresident aliens to sue in federal court. Answer (C) is incorrect because the relatives of the deceased have sufficient standing to bring a wrongful death action under Article III.

Congress recently passed a federal act aimed at keeping children from smoking. The act is a comprehensive federal law that regulates the marketing, distribution, and sale of tobacco products to minors. The law bans all tobacco advertising that targets minors, bans all sales of tobacco products for someone under the age of 18. The law also requires retail stores that sell tobacco products to follow stringent age verification procedures when selling tobacco products to persons who appear underage. One subsection of the law makes it a federal crime punishable by up to $1,000 fine to possess any tobacco product in or near a "child-related facility". The act defined "child-related facility" to include (among other things) schools, daycare facilities, children's hospitals, and orphanages. Congress made findings that (1) children and teenagers often obtain tobacco from adults and (2) the presence of tobacco products in child-friendly settings socializes children to believe that tobacco use is fun and safe, thereby undermining the act's ban on advertising to minors. Congress relied on its power under the Commerce Clause in passing the act. A father was recently arrested and indicted for violating the act by smoking a cigarette during the half-time of his child's high school basket ball game. In his defense, the father argues that the act exceeds Congress' power under the Commerce Clause. The father's argument is: (A) Correct because Congress may not regulate an intra-state non-economic activity (B) Correct because smoking near children is a non-economic activity with too remote a connection to interstate commerce (C) Incorrect because "child-related facilities" are within the channels of interstate commerce (D) Incorrect because Congress rationally concluded that regulation of tobacco possession in or near a child-related facility is necessary to the success of Congress' larger statutory scheme regulation the marketing, distribution and sale of tobacco products to minors

(D) Incorrect because Congress rationally concluded that regulation of tobacco possession in or near a child-related facility is necessary to the success of Congress' larger statutory scheme regulation the marketing, distribution and sale of tobacco products to minors Answer (D) is correct because the question asks that you apply Gonzales v. Raich, which held that the Commerce Clause reaches regulation of intra-state, non-economic activity when Congress has a rational basis for concluding that such regulation is necessary to the successful regulation of a larger interstate economic activity. Gonzales v. Raich, 545 U.S. 1 (2005)

Congress recently enacted an act regarding providing legal services to the poor. The act establishes a poverty law office in every state that is responsible for representing indigent clients in family law matters, such as divorce, child custody and child support matters. The act funds these new offices by imposing an occupation tax on every attorney licensed to practice in a state, territory, or possession of the United States. To facilitate collection of the attorney occupation tax, the act requires every firm or other organization, public or private, that employs lawyers to register with the Department of Labor, and to report quarterly the names and wages of each lawyer then employed. The act places a similar reporting requirement on solo practitioners. A national association of state attorney generals recently spoke out against the law, arguing that the act's registration requirement and occupation tax violate the residue of state sovereignty protected by the Constitution and Bill of Rights when applied to attorneys employed by the state. Does the act's attorney occupation tax violate the states' intergovernmental tax immunity? (A) Yes, because the act impermissibly taxes the states (B) Yes, because the act impermissibly taxes attorneys engaged in the traditional government function of law enforcement (C) No, because the act taxes state employees and not the state itself (D) No, because the act imposes a nondiscriminatory tax on those who deal with the state

(D) No, because the act imposes a nondiscriminatory tax on those who deal with the state Answer (D) is correct because while the federal government is generally prohibited from collecting a tax directly from the states, here the act imposes a tax on third parties who have dealt with the state--specifically, lawyers employed by state--instead of collecting the tax directly from state coffers. Answer (C) is incorrect because it mistakenly assumes that Congress may impose a tax on third parties regardless of whether the tax is discriminatory but Baker makes clear that discriminatory taxes are invalid. South Carolina v. Baker, 485 U.S. 505 (1988)

A state recently enacted a law prohibiting the importation and disposal of hazardous waste from outside the state. A disposal corporation is in the business of transporting hazardous waste across state lines, including carriage of waste into and out of the state. The state attorney general has sent the corporation a letter directing it to cease and desist importation of hazardous waste into the state. Believing that the state's import ban violates the Dormant Commerce Clause, the corporation filed suit against the state attorney general in federal court. The corporation's suit seeks an injunction against enforcement of the state's hazardous waste import ban. The state attorney general moves to dismiss the corporation's suit for lack of justicability. Should the court grant the attorney general's motion? (A) Yes, because the corporation does not have standing (B) Yes, because the corporation's claim is not ripe (C) Yes, because the Eleventh Amendment bars this suit against a state official (D) No, because there is no constitutional bar to the corporation's suit in federal court

(D) No, because there is no constitutional bar to the corporation's suit in federal court Answer (D) is correct because none of the other arguments prevent a federal court from hearing the corporation's lawsuit Ex Parte Young, 209 U.S.123 (1908)

After a recent newspaper article disclosed its existence, the President announced that the United States National Security Agency has been operating a domestic foreign intelligence surveillance program without seeking prior judicial approval. The surveillance applies to communications into and outside the United States where at least one of the parties is known or suspected to have links to terrorism. Suspicion can be related to a person's country of citizenship or prior activities. A lawyer who represents clients in international financial transactions and advises them on compliance with United States anti-money laundering and foreign trade laws recently filed suit against the NSA for an injunction declaring the secret surveillance plan unconstitutional. The lawyer argues that the surveillance program violates both the Fourth Amendment and the separation of powers. In support of his standing, the lawyer claims that soon after the President announced the existence of the NSA's secret surveillance program, several of his clients terminated his representation. He further alleges that these clients all resided or did business in countries the President had identified as harboring suspected terrorists. The NSA's strongest argument to dismiss this lawsuit for lack of standing is: (A) The lawyer has not alleged an actual injury (B) The lawyer has alleged a generalized injury shared by a large portion of the United States population (C) The lawyer's alleged injury does not have a sufficient nexus with the alleged constitutional violations (D) The causation and redressability connections between the lawyer's alleged injury and the NSA program are too speculative

(D) The causation and redressability connections between the lawyer's alleged injury and the NSA program are too speculative Answer (D) is correct because the Supreme Court has held that standing fails if it is speculative that either the government's challenged action caused the plaintiff's alleged harm, or that a judicial remedy will redress the harm. Here, there is a strong argument that the lawyer's claims of causation and redress are speculative. On the given facts, it is not clear why the lawyer is losing his international clients or if enjoining the program will cause those clients to return.

A state has passed legislation making it more difficult for employers to leave the state. The law requires employers to pay retirement benefits to certain employees even when the benefits have yet to vest under an employer's retirement plan. For example, under the law, an employer moving its entire operation out of state must pay a "pension funding charge" to cover the retirement benefits due any employee who has worked for employer for at least ten years, even if the employee's right to retirement benefits has not vested in this ten-year period. The state has not previously regulated retirement benefits offered by private employers. A company seeks legal advice as to whether this new state law violates the Contract Clause. (A) The company will be told that a claim under the Contract Clause is unlikely to succeed considering the Supreme Court's decisions interpreting the Clause in the last century (B) The company will be told that a claim under the Contract Clause is unlikely to succeed because it will be dismissed under the Eleventh Amendment (C) The company will be told that a claim under the Contract Clause can only be made against the federal government rather than a state government (D) The company will be told that a claim under the Contract Clause is likely to succeed because the state did not pass the law to deal with a broad economic problem, the law was totally unanticipated by the company, and the law is not temporary

(D) The company will be told that a claim under the Contract Clause is likely to succeed because the state did not pass the law to deal with a broad economic problem, the law was totally unanticipated by the company, and the law is not temporary Answer (D) is correct because it is in accord with the Court's decision in Allied Structural Steel Co. v. Spannaus, 438 U.S. 234 (1978) which is almost identical to the fact pattern. The Court held that the state law violated the Contract Clause--focusing on the fact that it was not emergency legislation and the law was not "enacted to protect a broad societal interest rather than a narrow class". Further, the Court emphasized that the state had not previously regulated this area, so employers could not have reasonably expected the law and "... relied heavily, and reasonably, on this legitimate contractual expectation in calculating its annual contributions to the pension fund". While Allied Structual Steel has never been overruled but it has also never been followed by the Court in any other case

Congress recently enacted an infant protection law aimed at preventing the switching of newborns. The act provides: Whoever being in interstate commerce knowingly alters or destroys an identification record of a newborn patient with the intention that the newborn patient be misidentified by any person shall be fined not more than $250,000 in the case of an individual and not more than $500,000 in the case of an organization, or imprisoned not more than ten years, or both. Can Congress constitutionally apply such a statute to a state hospital? (A) No, because Congress may not commandeer state legislatures into regulating state hospitals' handling of newborns (B) No, because Congress may not commandeer state executive branch officials into enforcing a federal regulatory scheme (C) No, because the Commerce Clause does not grant Congress power to regulate states (D) Yes, because Congress is regulating the state on the same terms that it regulates private actors

(D) Yes, because Congress is regulating the state on the same terms that it regulates private actors Answer (D) is correct because federal law commandeers only if it directs either the state legislature or other state officials to exercise their sovereign power over private actors. Here, the state will need to take some actions to comply with the act; however, none of these actions entail sovereign action against private citizens. Answer (C) is wrong because the Condon Court noted that its decision in Garcia v. San Antonio Metro. Trans. Auth., 469 U.S. 528 (1985) held that Congress may regulate states on the same terms as it does private actors.

The Clean Air and Water Protection Act is a federal statute enacted as part of a comprehensive program to protect the air and water quality of the United States. As an inseverable component of the legislation, a committee of five members was given the power to draft and enforce regulations to regulate air and water quality. According to the statute, three of the five members were to be appointed by the President and other two committee members would be selected by the top official in charge of the federal Environmental Protection Agency. Behemoth, Inc., a major chemical company, brought an action in the federal district court claiming that the statute was unconstitutional. The federal court should rule that the statute is: (A) unconstitutional, because the members of the committee will be exercising administrative power and thus can only be appointed by the President (B) unconstitutional, because the members of the committee will be exercising quasi-legislative power and thus can only be appointed by Congress (C) constitutional, because Congress may delegate to committees the power to draft whatever legislation may be necessary and proper to effectuate valid federal legislative goals (D) constitutional, because Congress may vest the appointment of inferior officers in this manner

(D) constitutional, because Congress may vest the appointment of inferior officers in this manner Answer (D) is correct. Under the Appointment Clause of Article II, Congress decides who appoints inferior officers: (a) the President, (b) heads of Departments, or (c) Courts of law--or any combination thereof. Here, Congress vested the power to appoint three inferior officers to the President and two inferior officers to the head of the EPA, therefore this scheme is constitutional under the Appointment Clause (Buckley v. Vadeo, 424 U.S. 1 (1976))


संबंधित स्टडी सेट्स

CITI Social & Behavioral Research-basic/refresher

View Set

research in growth & motor development

View Set

Ch 7 Settings for Development: Home, School, and Community

View Set

K - Unit 1 - Common Greetings and Classroom Directions

View Set

week 6 GI/Elmination stoma/enteral

View Set

Chapter 3 Human Anatomy and Physiology

View Set

Chapter 10 (Strategic Decision-Making)

View Set